Chương 1: KHỐI ĐA DIỆN

Hoàng Thị Tâm
Xem chi tiết
Bùi Bích Phương
1 tháng 4 2016 lúc 12:46

Hai mặt phẳng (SAB) và (SAC) cùng vuông góc với (ABC) \(\Rightarrow SA\perp\left(ABC\right)\)

\(AB\perp BC\Rightarrow SB\perp BC\Rightarrow\widehat{SBA}\) là góc giữa 2 mặt phẳng (SBC) và mặt phẳng (ABC)

\(\Rightarrow\widehat{SBA}=60^o\)

\(\Rightarrow SA=AB.\tan\widehat{SBA}=2a\sqrt{3}\)

Mặt phẳng qua SM và song song với BC, cắt AC tại N

\(\Rightarrow MN||BC\) và N là trung điểm của \(AC\\ \)

\(MN=\frac{BC}{2}=a;BM=\frac{AB}{2}=a\)

Diện tích \(S_{BCNM}=\frac{\left(BC+MN\right).BM}{2}=\frac{3a^2}{2}\)

Thể tích \(V_{S.BCNM}=\frac{1}{3}S_{BCNM}.SA=a^3\sqrt{3}\)

Kẻ đường thẳng \(\Delta\) đi qua N, song song với AB

Hạ \(AD\perp\Delta\left(D\in\Delta\right)\Rightarrow AB||\left(SND\right)\)

                                 \(\Rightarrow d\left(AB;SN\right)=d\left(AB,\left(SND\right)\right)=d\left(A,\left(SND\right)\right)\)

Hạ \(AH\perp SD\left(H\in SD\right)\Rightarrow AH\perp\left(SND\right)\Rightarrow d\left(A,\left(SND\right)\right)=AH\)

Tam giác SAD vuông tại A : \(\begin{cases}AH\perp SD\\AD=MN=a\end{cases}\)

                                            \(\Rightarrow d\left(AB,SN\right)=AH=\frac{SA.AD}{\sqrt{SA^2+AD^2}}=\frac{2a\sqrt{39}}{13}\)

 

Bình luận (0)
Thiên Di
31 tháng 3 2016 lúc 22:17

1242

 

Bình luận (0)
Bùi Bích Phương
1 tháng 4 2016 lúc 12:52

S A M B C N H D

Bình luận (0)
Lại Thị Hồng Liên
Xem chi tiết
Phạm Thái Dương
31 tháng 3 2016 lúc 9:29

x s K A N B H D C

Ta có : \(\widehat{SCH}\) là góc giữa SC và mặt phẳng (ABC). 

\(\Rightarrow\widehat{SCH}=60^0\)

Gọi D là trung điểm cạnh AB. Ta có :

\(HD=\frac{a}{6}\), CD= \(\frac{a\sqrt{3}}{2}\)

\(HC=\sqrt{HD^2+CD^2}=\frac{a\sqrt{7}}{3}\)

\(SH=HC.\tan60^0=\frac{a\sqrt{21}}{3}\)

\(V_{s.ABC}=\frac{1}{3}.SH.S_{\Delta ABC}=\frac{1}{3}.\frac{a\sqrt{21}}{3}.\frac{a^2\sqrt{3}}{4}=\frac{a^3\sqrt{7}}{12}\)

Kẻ Ax song song với BC, gọi N, K lần lượt là hình chiếu vuông góc của H lên Ax và SN. Ta có BC song song với mặt phẳng (SAN) và \(BA=\frac{3}{2}HA\)

Nên \(d\left(SA.BC\right)=d\left(B,\left(SAN\right)\right)=\frac{3}{2}d\left(H.\left(SAN\right)\right)\)

\(AH=\frac{2a}{3}\)\(HN=AH.\sin60^0=\frac{a\sqrt{3}}{3}\)

\(HK=\frac{SH.HN}{\sqrt{SH^2+HN^2}}=\frac{a\sqrt{42}}{12}\)

Vậy \(d\left(SA.BC\right)=\frac{a\sqrt{42}}{8}\)

Bình luận (4)
Thiên Thảo
30 tháng 3 2016 lúc 19:47

Góc 60 là góc SCH. Dễ dàng tính được V
Trong (ABC), kẻ At // BC, Cz//AB, giao At=N
d(sa,bc)=d(bc, (SAN))=d(B, (SAN))=3/2 d(H, (SAN)).
Từ H kẻ HE vuông AN
 Trong (SHE) kẻ HF vuông SE
=> d(H(SAN))=HF

Bình luận (0)
Phạm Yến Linh Linh
Xem chi tiết
Akai Haruma
15 tháng 8 2017 lúc 0:33

Lời giải:

Tính toán đơn giản: \(AC=\sqrt{3}a, DB=a\)

Ý 1:

Do \(SA\perp (ABCD)\Rightarrow SA\perp AC\). Áp dụng định lý Pitago:

\( \frac{1}{d(A,SC)^2}=\frac{1}{SA^2}+\frac{1}{AC^2}\Leftrightarrow \frac{1}{a^2}=\frac{1}{SA^2}+\frac{1}{3a^2}\Rightarrow SA=\frac{\sqrt{6}}{2}a\)

\(\Rightarrow V_{\text{chóp}}=\frac{1}{3}.SA.S_{ABCD}=\frac{1}{3}.\frac{\sqrt{6}a}{2}.\frac{AC.BD}{2}=\frac{\sqrt{2}a^3}{4}\)

Ý 2:

Kẻ \(AH\perp BC\) với \(H\in BC\). Có \(\left\{\begin{matrix} AH\perp BC\\ SA\perp BC\end{matrix}\right.\Rightarrow BC\perp (SAH)\)

Kẻ \(AT\perp SH\), mà \(AT\perp BC\) do \(AT\in (SAH)\) , do đó \(AT\perp (SBC)\)

\(\Rightarrow AT=d(A,(SBC))=\sqrt{\frac{SA^2.AH^2}{SA^2+AH^2}}\)

\(AH=\sin 60.AB=\frac{\sqrt{3}a}{2}\), suy ra \(d(A,(SBC))=AT=\frac{\sqrt{2}a}{2}\)

Ý 3:

Kẻ \(BK\parallel AC\) cắt $AD$ tại $K$

Ta có: \(d(SB,AC)=d(AC,(SBK))=d(A,(SBK))\)

Kẻ \(AR\perp BK\).

\(AR=AB.\sin ABK=AB.\sin BAC=AB\sin 30=\frac{a}{2}\)

Kẻ \(AM\perp SR\) thì $AM$ chính là khoảng cách từ $A$ đến $(SBK)$

\(d(A,(SBK))=AM=\sqrt{\frac{SA^2.AR^2}{SA^2+AR^2}}=\frac{\sqrt{42}a}{14}\)

Bình luận (0)
Mai Lê
Xem chi tiết
Thị Thanh Thảo Tô
Xem chi tiết
Hoàng Thị Thu Huyền
2 tháng 8 2017 lúc 14:59

Do hai khối chóp trên có chung chiều cao nên ta xét diện tích hai đáy. Xét hình vẽ sau khi tách mặt phẳng chứa đáy ABCD:

A B C D M N

Giả sử \(\dfrac{AD}{AN}=k\Rightarrow\dfrac{AB}{AM}=4-2k\), ĐK \(0< k< 2\)

Ta có \(\dfrac{S_{AMN}}{S_{ABCD}}=\dfrac{\dfrac{1}{2}AM.AN.sin\widehat{A}}{AB.AD.sin\widehat{A}}=\dfrac{1}{2}.\dfrac{1}{4-2k}.\dfrac{1}{k}=\dfrac{1}{4k\left(2-k\right)}\)

Ta thấy rằng \(\dfrac{V_1}{V}=\dfrac{S_{MBCDN}}{S_{ABCD}}=1-\dfrac{S_{AMN}}{S_{ABCD}}\)

Vậy \(\dfrac{V_1}{V}\) max khi \(\dfrac{1}{4k\left(2-k\right)}\) min

Với 0 < k < 2 thì \(min\dfrac{1}{4k\left(2-k\right)}=\dfrac{1}{4}\) khi k = 1

Vậy \(max\dfrac{V_1}{V}=\dfrac{3}{4}\) khi AN = AD và M là trung điểm AB.

Bình luận (0)
Nguyễn Đức Đạt
Xem chi tiết
Nguyễn Hồng Anh
30 tháng 3 2016 lúc 22:14

A B D O H S C

Gọi D là trung điểm của cạnh AB và O là tâm của tam giác ABC.

Ta có \(\begin{cases}AB\perp CD\\AB\perp SO\end{cases}\) nên \(AB\perp\left(SCD\right)\)

Do đó \(AB\perp SC\)

Mặt khác \(SC\perp AH\) suy ra \(SC\perp\left(ABH\right)\)

Ta có : \(CD=\frac{a\sqrt{3}}{2};OC=\frac{a\sqrt{3}}{2}\) nên \(SO=\sqrt{SC^2-OC^2}=\frac{a\sqrt{33}}{3}\)

Do đó : \(DH=\frac{SO.CD}{SC}=\frac{a\sqrt{11}}{4}\Rightarrow S_{\Delta ABH}=\frac{1}{2}AB.DH=\frac{\sqrt{11}a^2}{8}\)

Ta có : \(SH=SC-HC=SC-\sqrt{CD^2-DH^2}=\frac{7a}{4}\)

Do đó : \(V_{S.ABH}=\frac{1}{3}SH.S_{\Delta ABH}=\frac{7\sqrt{11}a^3}{96}\)

Bình luận (10)
Thiên Thảo
30 tháng 3 2016 lúc 19:47

V(SABC) = SA.S(ABC)/3 = 2a.(a√3/2).a/6 = a^3√3/6 
gọi khoảng cách từ A đến mp(SBC) là h, ta có: 
V1 = V(SAMN) = V(ASMN) = S(SMN).h/3 
V = V(SABC) = V(ASBC) = S(SBC).h/3 
=> V1/V = S(SMN)/S(SBC) = 1/2.SM.SN.sin(MSN^)/1/2.SB.SC.sin(MSN^) = (SM/SB).(SN/SC) 
SB = SC (do AB = AC) và SM = SN ( = SA^2/SB) 
=> V1/V = (SM/SB)^2 
SB^2 = SA^2 + AB^2 = 4a^2 + a^2 = 5a^2 => SB = a√5 
SM = SA^2/SB = 4a^2/(a√5) = 4a/√5 
=> V1/V = (16a^2/5)/(5a^2) = 16/25 
=> (V - V1)/V = 9/25 
=> V(A.BCNM) = (V - V1) = 9.V/25 = 9.(a^3√3/6)/25 = 3a^3√3/50 

Bình luận (0)
Minh Ole
Xem chi tiết
Akai Haruma
8 tháng 1 2017 lúc 18:45

Lời giải:

(Mình không biết vẽ hình kg trên này, mong bạn thông cảm)

Trước tiên có \(BA=AM=BM=a,AC=\sqrt{3}a\)

Dễ thấy tam giác $BAM$ là tam giác đều, $SB=SM=SA$ nên $SBAM$ là hình chóp tam giác đều. Do đó chân đường cao hạ từ $S$ xuống mặt phẳng $(BAM)$ là trọng tâm của tam giác $BAM$. Đặt điểm này là $T$. Khi đó $ST$ cũng là đường cao của hình chóp $S.ABC$

Dễ thấy \(BT=\frac{\sqrt{3}a}{3};SB=\frac{\sqrt{39}a}{3}\)\(\Rightarrow ST=\sqrt{SB^2-BT^2}=2a\)

Đây chính là độ dài cần tìm

Bình luận (0)
Dao Nguyen
Xem chi tiết
Nguyễn Hoàng Việt
18 tháng 12 2016 lúc 17:34

a) Dễ dàng chứng minh tam giác ABC và ACD đều

Suy ra AC=a, SA= AC.tan(gócSCA)=a.tan(600)

\(V_{S.ABCD}=\frac{1}{3}.SA.S_{ABCD}=\frac{1}{3}.a\sqrt{3}.a^2.\frac{\sqrt{3}}{2}=\frac{a^3}{2}\)

b) Có 2 cách làm để tìm khoảng cách từ H đến mp(SCD), nhưng bạn nên chọn phương pháp tọa độ hóa cho dễ

Chọn A làm gốc tọa độ , các tia AD, AI, AS lần lượt trùng tia Ax, Ay, Az

Có ngay tọa độ các điểm \(S\left(0;0;a\sqrt{3}\right)\) , \(D\left(a;0;0\right)\) , \(I\left(0;\frac{a\sqrt{3}}{2};0\right)\)

\(\Rightarrow C\left(\frac{a}{2};\frac{a\sqrt{3}}{2};0\right)\)

theo số liệu đã cho, dễ xác định được điểm H chia đoạn SI với tỷ lệ 2:1

\(\Rightarrow H\left(0;\frac{a}{\sqrt{3}};\frac{a}{\sqrt{3}}\right)\)

Bây giờ chỉ cần viết pt (SCD) là tính được ngay khoảng cách từ H đến SCD

\(\left(SCD\right):\sqrt{3}x+y+z-\sqrt{3}=0\)

\(d\left(H\text{/}\left(SCD\right)\right)=\frac{a\sqrt{3}}{\sqrt{5}}\)

Bình luận (5)
Võ Thị Hoài Linh
Xem chi tiết
Đào Thị Hương Lý
29 tháng 3 2016 lúc 22:05

A B C S H

Gọi H là trung điểm của BC=> HA=HB=HC

Kết hợp với giả thiết

SA=SB=SC=>\(SH\perp BC,\Delta SHA=\Delta SHB=SHC\)

\(\begin{cases}SH\perp\left(ABC\right)\\\widehat{SAH}=60^0\end{cases}\)

Tam giác ABC là tam giác vuông cân tại A

\(AC=AB=a\sqrt{2}\Rightarrow BC=2a\Rightarrow AH=a\)

Tam giác SHA vuông :

\(SH=AH.\tan60^0=a\sqrt{3}\Rightarrow V_{S.ABC}=\frac{1}{3}.\frac{1}{2}AB.AC.SH=\frac{\sqrt{3}a^3}{3}\)

Gọi O; R lần lượt là tâm và bán kính của mặt cầu ngoại tiếp chóp S.ABC. Suy ra P thuộc đường thẳng SH, nên O thuộc mặt phẳng (SBC). Do đó R là bán kính đường tròn ngoại tiếp tam giác SBC. 

Xét tam giác SHA ta có : \(SA=\frac{SH}{\sin60^0}=2a\Rightarrow\Delta SBC\) là tam giác đều có độ dài cạnh bằng 2a.

Suy ra \(R=\frac{2a}{2\sin60^0}=\frac{2a\sqrt{3}}{3}\)

Bình luận (0)
Nguyễn Bình Nguyên
Xem chi tiết
Lương Đức Trọng
1 tháng 4 2016 lúc 16:24

?o?n th?ng a: ?o?n th?ng [A, D] ?o?n th?ng b: ?o?n th?ng [A, B] ?o?n th?ng e: ?o?n th?ng [B, C] ?o?n th?ng f: ?o?n th?ng [C, D] ?o?n th?ng g: ?o?n th?ng [A, C] ?o?n th?ng i: ?o?n th?ng [S, H] ?o?n th?ng j: ?o?n th?ng [S, A] ?o?n th?ng k: ?o?n th?ng [S, B] ?o?n th?ng l: ?o?n th?ng [S, C] ?o?n th?ng m: ?o?n th?ng [S, D] ?o?n th?ng n: ?o?n th?ng [M, C] ?o?n th?ng p: ?o?n th?ng [M, B] ?o?n th?ng p: ?o?n th?ng [M, B] A = (-1.48, 1.8) A = (-1.48, 1.8) A = (-1.48, 1.8) D = (2.3, 1.8) D = (2.3, 1.8) D = (2.3, 1.8) B = (-3.12, -0.08) B = (-3.12, -0.08) B = (-3.12, -0.08) ?i?m C: Giao ?i?m c?a c, d ?i?m C: Giao ?i?m c?a c, d ?i?m C: Giao ?i?m c?a c, d ?i?m H: (3A + C) / 4 ?i?m H: (3A + C) / 4 ?i?m H: (3A + C) / 4 ?i?m S: ?i?m tr�n h ?i?m S: ?i?m tr�n h ?i?m S: ?i?m tr�n h ?i?m M: (S + A) / 2 ?i?m M: (S + A) / 2 ?i?m M: (S + A) / 2

Do CM là trung tuyến của SAC nên M là trung điểm SA.

\(\dfrac{V_{SMBC}}{V_{SABC}}=\dfrac{SM}{SA}=\dfrac{1}{2}\)

Ta có \(AC=\sqrt{AB^2+AC^2}=a\sqrt{2}\) nên \(AH=\dfrac{1}{4}AC=\dfrac{a\sqrt{2}}{4}\)

Suy ra \(SH=\sqrt{SA^2-AH^2}=\sqrt{a^2-\dfrac{2a^2}{16}}=\dfrac{a\sqrt{14}}{4}\)

Do đó \(V_{SABC}=\dfrac{1}{3}SH.S_{ABC}=\dfrac{1}{3}.\dfrac{a\sqrt{14}}{4}.\dfrac{a^2}{2}=\dfrac{a^3\sqrt{14}}{24}\)

Vậy \(V_{SMBC}=\dfrac{1}{2}V_{SABC}=\dfrac{a^3\sqrt{14}}{48}\)

 

 

Bình luận (1)